IMO-1967 ll-35

Discussion on International Mathematical Olympiad (IMO)
User avatar
SANZEED
Posts:550
Joined:Wed Dec 28, 2011 6:45 pm
Location:Mymensingh, Bangladesh
IMO-1967 ll-35

Unread post by SANZEED » Wed Sep 19, 2012 6:05 am

Prove that for arbitrary positive integers $a,b,c$ the following holds:
$\frac{1}{a}+\frac{1}{b}+\frac{1}{c}\leq \frac{a^{8}+b^{8}+c^{8}}{a^{3}b^{3}c^{3}}$
P.S.
I can present a solution here.Is it correct?
Multiplying both sides with $a^{4}b^{4}c^{4}$ the inequality becomes
$\displaystyle\sum_{sym}a^{4}b^{4}c^{3}\leq \displaystyle\sum_{sym}a^{9}bc$.
According to Muirhead's theorem, it suffices to prove that $6[4,4,3]\leq 6[9,1,1]$ which is true by Muirhead if I am not mistaken.Am I correct?
$\color{blue}{\textit{To}} \color{red}{\textit{ problems }} \color{blue}{\textit{I am encountering with-}} \color{green}{\textit{AVADA KEDAVRA!}}$

Post Reply